• Anúncio Global
    Respostas
    Exibições
    Última mensagem

ajuda urgente

ajuda urgente

Mensagempor fna » Sex Mai 24, 2013 03:28

Se a=2^3,b=a^2 e c=2^2a,então o valor de 8abc eh:
A resposta eh 4^14 como chego a esse valor?
fna
Usuário Ativo
Usuário Ativo
 
Mensagens: 16
Registrado em: Qua Abr 24, 2013 04:22
Formação Escolar: ENSINO MÉDIO
Área/Curso: vestibular
Andamento: cursando

Re: ajuda urgente

Mensagempor Rafael16 » Sex Mai 24, 2013 12:37

a=2^3, b=a^2 e c = 2^{2a}.

8abc =

8.2^3.a^2.2^{2a}=

8.8.(2^3)^2.2^{2.2^3}=

8.8.2^6.2^{2.8}=

8.8.2^6.2^{16}=

2^3 . 2^3.2^6 . 2^{16}=

2^{28}=

2^{14}. 2^{14}

4^{14}
Rafael16
Colaborador Voluntário
Colaborador Voluntário
 
Mensagens: 154
Registrado em: Qui Mar 01, 2012 22:24
Formação Escolar: GRADUAÇÃO
Área/Curso: Análise de Sistemas
Andamento: cursando


Voltar para Álgebra Elementar

 



  • Tópicos relacionados
    Respostas
    Exibições
    Última mensagem

Quem está online

Usuários navegando neste fórum: Nenhum usuário registrado e 2 visitantes

 



Assunto: Unesp - 95 Números Complexos
Autor: Alucard014 - Dom Ago 01, 2010 18:22

(UNESP - 95) Seja L o Afixo de um Número complexo a=\sqrt{8}+ i em um sistema de coordenadas cartesianas xOy. Determine o número complexo b , de módulo igual a 1 , cujo afixo M pertence ao quarto quadrante e é tal que o ângulo LÔM é reto.


Assunto: Unesp - 95 Números Complexos
Autor: MarceloFantini - Qui Ago 05, 2010 17:27

Seja \alpha o ângulo entre o eixo horizontal e o afixo a. O triângulo é retângulo com catetos 1 e \sqrt{8}, tal que tg \alpha = \frac{1}{sqrt{8}}. Seja \theta o ângulo complementar. Então tg \theta = \sqrt{8}. Como \alpha + \theta = \frac{\pi}{2}, o ângulo que o afixo b formará com a horizontal será \theta, mas negativo pois tem de ser no quarto quadrante. Se b = x+yi, então \frac{y}{x} = \sqrt {8} \Rightarrow y = x\sqrt{8}. Como módulo é um: |b| = \sqrt { x^2 + y^2 } = 1 \Rightarrow x^2 + y^2 = 1 \Rightarrow x^2 + 8x^2 = 1 \Rightarrow x = \frac{1}{3} \Rightarrow y = \frac{\sqrt{8}}{3}.

Logo, o afixo é b = \frac{1 + i\sqrt{8}}{3}.